Đến nội dung

dactai10a1 nội dung

Có 275 mục bởi dactai10a1 (Tìm giới hạn từ 04-06-2020)



Sắp theo                Sắp xếp  

#439006 Chứng minh OH=OG

Đã gửi bởi dactai10a1 on 29-07-2013 - 01:05 trong Hình học

Ký hiệu như hình vẽ.

ScreenHunter_02%20Jul.%2028%2023.27.gif

 

1. Dễ chứng minh được $I, N, F$ thẳng hàng; $O, E, N$ thẳng hàng.

2. Theo Định lý Brocard, ta có $IO$ vuông góc $FG$ và $OJ$ vuông góc $JF.$

3. Các cặp tam giác $JCD, JHG; JCP, JHN; JPD, JNG$ đồng dạng.

4. Từ đó $NH=NG.$

5. Vậy $OH=OG.$

câu a:vẫn làm được trên nền ý tưởng của thầy

chỉ khác chỗ phần 3:cap tam giac JCA va JHG đồng dạng.P là trung điểm của AC 




#438848 Chứng minh OH=OG

Đã gửi bởi dactai10a1 on 28-07-2013 - 14:16 trong Hình học

Cho tứ giác ABCD nội tiếp đường tròn (O).AB cắt CD tại F.AC cắt BD tại E,

a)Đường thẳng FE cắt 2 đường tròn (EBC) và (EAD) lần lượt tại H,G.Chứng minh OH=OG

b)Đường thẳng FE cắt 2 đường tròn (FBC) và (FAD) lần lượt tại H,G.Chứng minh OH=OG




#432560 $x_1^4 + x_2^4 + ... + x_n^4 = 1998$

Đã gửi bởi dactai10a1 on 03-07-2013 - 17:08 trong Số học

Nhận xét rằng $a^{4}\equiv 0;1(mod16)$

Áp dụng vào bài toán ta có : $VT\equiv 0;1;2;....;n$ $(mod 16)$

Mà $VP=1998\equiv 14(mod16)$

Do đó để phương trình tồn tại nghiệm nguyên thì $n_{min}=14$

Không tồn tại 14 số nguyên dương mà tổng luỹ thừa =14,đáp số là 15 

\[1998 = {4^4} + {5^4} \times 2 + {3^4} \times 6 + 6 \times {1^4}\]




#432546 $x_1^4 + x_2^4 + ... + x_n^4 = 1998$

Đã gửi bởi dactai10a1 on 03-07-2013 - 16:08 trong Số học

Tìm số nguyên dương n nhỏ nhất sao cho tồn tại n số nguyên dương thoả $x_1^4 + x_2^4 + ... + x_n^4 = 1998$

 




#429789 CHứng minh X,Y,Z,T đồng viên hoặc thẳng hàng

Đã gửi bởi dactai10a1 on 22-06-2013 - 16:02 trong Hình học

Cho tứ giác lồi ABCD.AC giao BD tại O.Đường tròn (AOB) và (DOC) giao nhau tại X.Đường tròn (AOD) và (BOC) giao nhau tại Y.Đường tròn đường kính AC và đường tròn đường kính BD giao nhau tại Z và T.CHứng minh X,Y,Z,T đồng viên hoặc thẳng hàng 




#429458 ${a^3} + 16{b^2} = {a^2} + 18{b^3...

Đã gửi bởi dactai10a1 on 21-06-2013 - 11:24 trong Bất đẳng thức - Cực trị

Cho a,b>0 thoả ${a^3} + 16{b^2} = {a^2} + 18{b^3}$.Tìm min $P = a + b + \frac{6}{{ab}}$

 

 




#425309 ${x^3} - 2{y^3} - xy(x + y) - 2y + x = 0$

Đã gửi bởi dactai10a1 on 09-06-2013 - 10:18 trong Phương trình - Hệ phương trình - Bất phương trình

Giải hệ : $\left\{\begin{matrix} (x - 1)\left( {2\sqrt {2y - 1} + 3\sqrt[3]{{x + 6}}} \right) = 2y + 6\\ {x^3} - 2{y^3} - xy(x + y) - 2y + x = 0 \end{matrix}\right.$

 

-------------

 

@mod : 271 bài rồi mà vẫn chưa biết rành $Latex$,đặt sai tiêu đề, xứng đáng 1 nhắc nhở.




#425062 CMR $\frac{2MN}{EF}=\left|\frac{...

Đã gửi bởi dactai10a1 on 08-06-2013 - 15:45 trong Hình học

Cho tứ giác $ABCD$ nội tiếp.Gội $M$,$N$ lần lượt là trung điểm $AC$ và $BD$.$E$,$F$ lần lượt là giao của $AB$ với $CD$ , $AD$ với $BC$

CMR $\frac{2MN}{EF}=\left|\frac{AC}{BD}-\frac{BD}{AC}\right|$.

NGUỒN :ĐỀ KIỂM TRA ĐỘI DỰ TUYỂN IMO 2013

Đặt $\widehat {AEB} = \gamma ,EC = c,ED = d,\overrightarrow i = \frac{{\overrightarrow {EC} }}{c},\overrightarrow j = \frac{{\overrightarrow {ED} }}{d}$
Vì ABCD nội tiếp nên $\frac{{AB}}{{CD}} = \frac{{AE}}{{CE}} = \frac{{BE}}{{DE}} = k$
Suy ra $\overrightarrow {EA} = kc\overrightarrow j ;\overrightarrow {EB} = kd\overrightarrow i $
Vì F thuộc AC và BD nên tồn tại cặp số x,y thỏa mãn
$\left\{ {\begin{array}{*{20}{c}}

{\overrightarrow {{\rm{EF}}}  = x\overrightarrow {EA}  + (1 - x)\overrightarrow {EC}  = xkc\vec j + (1 - x)c\vec i}\\
{\overrightarrow {{\rm{EF}}}  = y\overrightarrow {EB}  + (1 - y)\overrightarrow {ED}  = ykd\vec i + (1 - y)d\vec j}
\end{array}} \right.$

 

Do $\overrightarrow i ;\overrightarrow j $ khác phương nên xkc=(1-y)d và ykd=(1-x)c. Suy ra $x = \frac{{kd - c}}{{({k^2} - 1)c}}$
Vậy $\overrightarrow {{\rm{EF}}} = \frac{k}{{{k^2} - 1}}((kd - c)\overrightarrow j + (kc - d)\overrightarrow i ) \Rightarrow {\rm{E}}{{\rm{F}}^2} = {\left( {\frac{k}{{{k^2} - 1}}} \right)^2}\left[ {{{(kd - c)}^2} + {{(kc - d)}^2} + 2(kd - c)(kc - d)c{\rm{os}}\gamma } \right]$
Mặt khác ta có $\overrightarrow {MN} = \frac{{\overrightarrow {AD} + \overrightarrow {BC} }}{2} = \frac{{\overrightarrow {ED} - \overrightarrow {EA} + \overrightarrow {EC} - \overrightarrow {EB} }}{2} = \frac{{(d - kc)\overrightarrow j + (c - dk)\overrightarrow j }}{2} \Rightarrow M{N^2} = \frac{{{{(d - kc)}^2} + {{(c - dk)}^2} + 2(d - kc)(c - dk)c{\rm{os}}\gamma }}{4}$
Từ đây ta có $\frac{{M{N^2}}}{{{\rm{E}}{{\rm{F}}^2}}} = \frac{1}{4}.{\left( {\frac{{{k^2} - 1}}{k}} \right)^2} = \frac{1}{4}{\left( {k - \frac{1}{k}} \right)^2} \Rightarrow \frac{{MN}}{{{\rm{EF}}}} = \frac{1}{2}\left| {k - \frac{1}{k}} \right| = \frac{1}{2}\left| {\frac{{AB}}{{CD}} - \frac{{CD}}{{AB}}} \right|$
Suy ra đpcm




#424950 ${x_1} = 1;{x_2} = - 1;{x_{n + 2}...

Đã gửi bởi dactai10a1 on 07-06-2013 - 22:27 trong Dãy số - Giới hạn



Bạn tính lại đi nhé! $x_2=-1$ chứ không phải $1$

${x_3} = {x_2}^2 - \frac{{{x_1}}}{2} = 1 - \frac{1}{2} = \frac{1}{2};{x_4} = x_3^2 - \frac{{{x_2}}}{2} = \frac{1}{4} - \frac{{ - 1}}{2} = \frac{1}{4} + \frac{1}{2} = \frac{3}{4}???$

 




#424893 ${x_1} = 1;{x_2} = - 1;{x_{n + 2}...

Đã gửi bởi dactai10a1 on 07-06-2013 - 20:17 trong Dãy số - Giới hạn

Bài này liệu có sai đề không nhỉ? Nếu không thì đơn giản quá!

Dễ tính được $2<x_4<x_5$

Quy nạp để chứng minh $x_{n+1}>x_n$ do $x_4>2$. Vậy $x_n$ là dãy tăng

Sau đó giả sử dãy có giới hạn hưũ hạn $L$ thì $L=L^2-\frac{L}{2}$ nên $L=0$ hoặc $L= \frac{3}{2}$ vô lí vì $x_n$ là dãy tăng và $x_4>2$.

Vậy $\lim x_n =+\infty$

 

x3=1/2;x4=3/4;x5=5/16 ?????

Lam sao qui nap $x_{n+1}>x_n$ ?????




#424483 ${x_1} = 1;{x_2} = - 1;{x_{n + 2}...

Đã gửi bởi dactai10a1 on 06-06-2013 - 15:22 trong Dãy số - Giới hạn

Cho $\left( {{x_n}} \right)$ thoả ${x_1} = 1;{x_2} =  - 1;{x_{n + 2}} = x_{n + 1}^2 - \frac{1}{2}{x_n}$.Tính $\mathop {\lim }\limits_{n \to \infty } {x_n}$

 

 

 




#421579 CHứng minh AL,BP,CQ đồng quy

Đã gửi bởi dactai10a1 on 27-05-2013 - 21:05 trong Hình học

Cho tam giác ABC không cân.Đường tròn nội tiếp (I) tiếp xúc với AC,AB tại F,E.Đường EF cắt BC tại S.Một đường thẳng qua S cắt (I) tại P và Q.EP cắt QF tại L.CHứng minh AL,BP,CQ đồng quy  




#421563 ${a_1} < {a_2} < ... < {a_n};...

Đã gửi bởi dactai10a1 on 27-05-2013 - 20:33 trong Số học

Trong bài toán này thì ta có 2 bộ số, mỗi bộ $n$ phần tử trừ đi nhau và có dấu giá trị tuyệt đối nên bản chất ở đây chính là lấy $n$ số lớn hơn trừ đi $n$ số bé hơn nên ta có:

$\sum_{i=1}^{n}\left |a_{i}-b_{i} \right |=[(n+1)+(n+2)+...+(n+n)]-[1+2+...+n]=n^{2}$

Bản giải thích kĩ lưỡng đi,chẳng hạn có truờng hợp ai=2;bi=n-1 thì sao




#420913 ${a_1} < {a_2} < ... < {a_n};...

Đã gửi bởi dactai10a1 on 25-05-2013 - 09:28 trong Số học

Các số 1,2,...2n-1,2n được chia thành 2 nhóm a_i và b_i,mỗi nhóm gồm n số .Giả sử ${a_1} < {a_2} < ... < {a_n};{b_1} > {b_2} > ... > {b_n}$.CMR $\left| {{a_1} - {b_1}} \right| + \left| {{a_2} - {b_2}} \right| + ... + \left| {{a_n} - {b_n}} \right| = {n^2}$

 

 

 




#416759 Chứng minh góc HMK bằng $\frac{1}{2}$ số đ...

Đã gửi bởi dactai10a1 on 05-05-2013 - 21:42 trong Hình học

CHo đường tròn $(O)$.Hai đường tròn $(C1)$ và $(C2)$ tiếp xúc trong với $(O)$ lần lượt tại $A$ và $F$.Hai đường tròn này cắt nhau tại 2 điểm D và E phân biệt.Gọi K là hình chiếu của A lên DE.H là trung điểm của AK.M là trung điểm $DE$.Chứng minh góc $HMK$ bằng $\frac{1}{2}$ số đo cung nhỏ $AF$ của $(O)$.




#416106 Chứng minh ${a^n} + {b^n} \in Z$

Đã gửi bởi dactai10a1 on 02-05-2013 - 21:45 trong Số học

Cho a,b là các số thực thoả ${a^n} + {b^n} \in Z$ với n=1,2,3,4.Chứng minh ${a^n} + {b^n} \in Z$ với mọi n




#416087 CHứng minh K,I,D,L đồng viên

Đã gửi bởi dactai10a1 on 02-05-2013 - 20:38 trong Hình học

Cho đường tròn (O) và 2 điểm A,B phân biệt thuộc (O).Lấy M là trung điểm AB.Một dây cung CD bất kì khác AB đi qua M.Gọi K là giao điểm của AC và BD.KM cắt (O) tại I và H(I nằm giữa H và K).AI cắt BH tại L.CHứng minh K,I,D,L đồng viên 




#413448 $n = \sum\limits_{i = 1}^n {{a_i}...

Đã gửi bởi dactai10a1 on 18-04-2013 - 20:12 trong Số học

Tìm tất cả số tự nhiên n sao cho tồn tại các số nguyên ${a_1};{a_2};...{a_n}$ thỏa $n = \sum\limits_{i = 1}^n {{a_i}} = {a_1}{a_2}...{a_n}$

n đồng dư với 0 hoặc 1 mod 4(n khác 4).

Với n=4k+1,chọn 2k số bằng -1;2k số bằng 1 và 1 số bằng n

Nếu n=4k,k chẵn,chọn k số bằng -1;3k-2 số bằng 1,1 số bằng 2,1 số bằng 2k

Nếu k lẻ.Chọn k-2 số bằng -1;3k số bằng 1,1 số bằng 2;1 số bằng 2k

Ta cm  n=4k+3 ko thỏa.Vì tích các a_i bằng n lẻ nên các số a_i đều lẻ.Mà tích đòng dư với -1 mod 4 nên có số lẻ các số a_i đòng dư -1 mod 4,chẳng hạn có 2m+1 số.Khi đó tổng đồng dư với (n-2m-1)-(2m+1)=n-2(mod 4).Vô lí

Nếu n=4k+2.Suy ra trong các số a_i có đúng 1 số chẵn.n-1 số còn lại đều lẻ nên tổng của n số này lẻ,mà n chẵn,vô lí  




#413270 $2^n-2 \vdots n$

Đã gửi bởi dactai10a1 on 17-04-2013 - 20:17 trong Số học

bài 2:
Giả sử m>2
Ta có \[{m^{{{(m - 1)}^2}}} \equiv {(1 + (m - 1))^{{{(m - 1)}^2}}} \equiv 1(\bmod {(m - 1)^2})\]

cho em hỏi làm sao để chọn mod như trên ạ ?

 

Ý tưởng là tìm 1 ví dụ phản chứng.Ta cần chọn n sao cho ${m^n} \equiv 1(\bmod n)$.Nhìn số 1 ta nghĩ ngay tới $m = m - 1 + 1 \Rightarrow {m^k} \equiv 1(\bmod m - 1)$(hệ thức không giống các định lí số học thông thường ).Thế nhưng $m \equiv 1(\bmod m - 1)$.Vì vậy ta cầnn nâng bậc lên để $m \ne 1(\bmod n)$( ta ko thêm hệ số vì ${m^n} \equiv 1(\bmod n)$ sẽ rất khó xử lí).Với $n = {(m - 1)^2}$ ta có ngay kết quả



#411956 $m^n\equiv 1(\mod n)$ thì $m\equiv 1(\mod...

Đã gửi bởi dactai10a1 on 11-04-2013 - 22:40 trong Số học

Tìm tất cả các số nguyên dương $m$ sao cho nếu tồn tại số nguyên dương $n$ thoả mãn $m^n\equiv 1(\mod n)$ thì $m\equiv 1(\mod n)$

http://diendantoanho...5-2n-2-vdots-n/




#410285 $p^q-q^p=pq^2-19 $

Đã gửi bởi dactai10a1 on 04-04-2013 - 09:27 trong Số học

  1. Cho các số nguyên $a,b,c$ và số nguyên tố $p$.CMR tồn tại $x,y,z$ không đồng thời chia hết cho $p$ sao cho $p|ax^2+by^2+cz^2$

 

Nếu có 1 trong 3 số a,b,c chia hết cho p,không mất tổng quát giả sử là a,thì khi đó ta chỉ cần chọn y=z=p; x=1

Nếu a,b,c đều không chia hết cho p

p=2.Dĩ nhiên thỏa

Xét p>2

Ta chọn trước z=1

Vì ${(p - k)^2} \equiv {k^2}(\bmod p)$nên với mọi số nguyên x thì${x^2}$ chia p chỉ có các số dư là ${0,1,2^2},...{\left( {\frac{{p - 1}}{2}} \right)^2}$

Xét các số${\rm{a}}{{\rm{x}}^2}$ (gọi là tập A)với x chạy qua$0,1,2,...\left( {\frac{{p - 1}}{2}} \right)$  và các số $ - b{y^2} - c$(gọi là tập B)với y chạy qua $0,1,2,...\left( {\frac{{p - 1}}{2}} \right)$

Dĩ nhiên ta có các số ${\rm{a}}{{\rm{x}}^2}$có số dư đôi một khác nhau khi chia cho p và các số$ - b{y^2} - c$ có số dư đôi một khác nhau khi chia cho p(*)

Tổng các phần tử của 2 tập A và B là p+1(mỗi tập có $\frac{{p + 1}}{2}$phần tử)

Mà 1 số khi chia cho p chỉ có p số dư.Kết hợp với (*) phải có 1 phần tử thuộc A và 1 phần tử thuộc B mà 2 phần tử này có cùng số dư khi chia cho p.Giả sử là ${\rm{ax}}_0^2; - by_0^2 - c$  

Khi đó ta có$ax_0^2 - ( - by_0^2 - c) \vdots p \Leftrightarrow ax_0^2 + by_0^2 + c \vdots p$

Vậy bộ số $\left( {{x_0};{y_0};1} \right)$thỏa đề bài.Suy ra đpcm




#410283 $(2005^{2005}-p^{2006})\vdots (2005+p)$

Đã gửi bởi dactai10a1 on 04-04-2013 - 09:12 trong Số học

Tìm số nguyên p sao cho $(2005^{2005}-p^{2006})\vdots (2005+p)$

p là số nguyên tố??http://diendantoanho...-20062a2007b2b/




#399839 $(m-n)^3\geq 3mn$

Đã gửi bởi dactai10a1 on 24-02-2013 - 23:34 trong Số học

Mấy bài của thầy Hà Duy Hưng
Bài toán 2: Cho $a>b$ nguyên dương thoả $a^2+ab+b^2|ab(a+b)$. Chứng minh rằng:

$(a-b)^3\geq 3ab$


Gọi $d = (a,b);d \in {N^*}$;Đặt $\left\{ {\begin{array}{*{20}{c}}
{a = dm}\\
{b = dn}
\end{array}(m,n \in {N^*};(m,n) = 1;m \ne n)} \right.$
Giả thiết tương đương với ${d^3}mn(m + n) \vdots {d^2}({m^2} + mn + {n^2}) \Leftrightarrow dmn(m + n) \vdots {m^2} + mn + {n^2}(1)$Gọi$h = (n;{m^2} + mn + {n^2})$ thì\[\left\{ {\begin{array}{*{20}{c}}
{n \vdots h}\\
{{m^2} + mn + {n^2} \vdots h}
\end{array} \Rightarrow \left\{ {\begin{array}{*{20}{c}}
{n \vdots h}\\
{m \vdots h}
\end{array}} \right.} \right.\]
Mà$(m,n) = 1$ nên h=1 hay $(n;{m^2} + mn + {n^2}) = 1$(2)
Tương tự $(m;{m^2} + mn + {n^2}) = 1$ (3)
Gọi k là 1 ước nguyên tố của n+m và ${m^2} + mn + {n^2}$ thì $\left\{ {\begin{array}{*{20}{c}}
{m + n \vdots k}\\
{{m^2} + mn + {n^2} \vdots k}
\end{array} \Leftrightarrow \left\{ {\begin{array}{*{20}{c}}
{{m^2} + 2mn + {n^2} \vdots k}\\
{{m^2} + mn + {n^2} \vdots k}
\end{array} \Leftrightarrow \left\{ {\begin{array}{*{20}{c}}
{mn \vdots k}\\
{m + n \vdots k}
\end{array}} \right.} \right.} \right. \Rightarrow \left\{ {\begin{array}{*{20}{c}}
{m \vdots k}\\
{n \vdots k}
\end{array}} \right.$ Mà (m,n)=1nên k=1 hay $(n + m;{m^2} + mn + {n^2}) = 1$ (4)
(1);(2);(3);(4) suy ra $d \vdots {m^2} + mn + {n^2}$ ma $d \in {N^*}$ nên $d \ge {m^2} + mn + {n^2} > mn$

Mà $m \ne n;m,n \in {N^*}$nên $\left| {m - n} \right| \ge 1$

\[ \Rightarrow {d^3}{\left| {m - n} \right|^3} > {d^2}mn \Leftrightarrow {\left| {a - b} \right|^3} > ab \Leftrightarrow \left| {a - b} \right| > \sqrt[3]{{ab}}\] SUy ra dpcm



#395535 Tìm các số nguyên x sao cho ${\left( {{x^2} - 3...

Đã gửi bởi dactai10a1 on 10-02-2013 - 19:39 trong Số học

Tìm các số nguyên x sao cho ${\left( {{x^2} - 3} \right)^2} + 12x$ là số chính phương



#389326 $A = \frac{{4a}}{{a + b + 2c}...

Đã gửi bởi dactai10a1 on 23-01-2013 - 18:57 trong Bất đẳng thức và cực trị

Cho a,b,c là các số thực dương.Tìm min $A = \frac{{4a}}{{a + b + 2c}} + \frac{{b + 3c}}{{2a + b + c}} - \frac{{8c}}{{a + b + 3c}}$